On continuous functions The Next CEO of Stack Overflowconvergence of $alpha$-Hölder-continuous functionsBasic calculus questionCheck my proof of the “Boundedness theorem”Show that $f$ is identically zero if $|f(x)|leqint_0^xf(t)dt$Proving set of bounded continuous functions is an open setFinding an appropriate continuous functionsIs there a bijective continuous function from closed to open intervalls?Difference between sup and inf of $|f(x)|$ less than difference between sup and inf of $f(x)$Giving examples for real functions with specific parametersSupremum metric on closed subset of continuous functions

Can you be charged for obstruction for refusing to answer questions?

Why isn't acceleration always zero whenever velocity is zero, such as the moment a ball bounces off a wall?

Is it professional to write unrelated content in an almost-empty email?

"misplaced omit" error when >centering columns

Math-accent symbol over parentheses enclosing accented symbol (amsmath)

Is wanting to ask what to write an indication that you need to change your story?

Which one is the true statement?

Is it possible to use a NPN BJT as switch, from single power source?

What was the first Unix version to run on a microcomputer?

Do I need to write [sic] when a number is less than 10 but isn't written out?

How many extra stops do monopods offer for tele photographs?

Writing differences on a blackboard

Why do airplanes bank sharply to the right after air-to-air refueling?

0 rank tensor vs 1D vector

Help understanding this unsettling image of Titan, Epimetheus, and Saturn's rings?

Domestic-to-international connection at Orlando (MCO)

Does Germany produce more waste than the US?

The past simple of "gaslight" – "gaslighted" or "gaslit"?

RigExpert AA-35 - Interpreting The Information

Where do students learn to solve polynomial equations these days?

What did we know about the Kessel run before the prequels?

Axiom Schema vs Axiom

Easy to read palindrome checker

Find non-case sensitive string in a mixed list of elements?



On continuous functions



The Next CEO of Stack Overflowconvergence of $alpha$-Hölder-continuous functionsBasic calculus questionCheck my proof of the “Boundedness theorem”Show that $f$ is identically zero if $|f(x)|leqint_0^xf(t)dt$Proving set of bounded continuous functions is an open setFinding an appropriate continuous functionsIs there a bijective continuous function from closed to open intervalls?Difference between sup and inf of $|f(x)|$ less than difference between sup and inf of $f(x)$Giving examples for real functions with specific parametersSupremum metric on closed subset of continuous functions










0












$begingroup$


Let $f:[0,1]to mathbb R$ be continuous and such that $f(0)<0$ and $f(1)=0$. Show that the set $x in (0,1] : f(x)=0 $ has a minimum.



So I though because the function is continuous, it is bounded. Since it is bounded it must have a inf and a sup. Since a inf exists in a bounded set I could achrimedian proof it is the minimum. But my professor told me that since the set he wants a minimum in is unbounded my logic wont work. So what am I doing wrong?










share|cite|improve this question











$endgroup$











  • $begingroup$
    The set is closed in $[0,1].$
    $endgroup$
    – Dbchatto67
    Mar 27 at 19:34










  • $begingroup$
    Not in the set im looking for a minimum in?
    $endgroup$
    – JJ Burt
    Mar 27 at 19:38















0












$begingroup$


Let $f:[0,1]to mathbb R$ be continuous and such that $f(0)<0$ and $f(1)=0$. Show that the set $x in (0,1] : f(x)=0 $ has a minimum.



So I though because the function is continuous, it is bounded. Since it is bounded it must have a inf and a sup. Since a inf exists in a bounded set I could achrimedian proof it is the minimum. But my professor told me that since the set he wants a minimum in is unbounded my logic wont work. So what am I doing wrong?










share|cite|improve this question











$endgroup$











  • $begingroup$
    The set is closed in $[0,1].$
    $endgroup$
    – Dbchatto67
    Mar 27 at 19:34










  • $begingroup$
    Not in the set im looking for a minimum in?
    $endgroup$
    – JJ Burt
    Mar 27 at 19:38













0












0








0





$begingroup$


Let $f:[0,1]to mathbb R$ be continuous and such that $f(0)<0$ and $f(1)=0$. Show that the set $x in (0,1] : f(x)=0 $ has a minimum.



So I though because the function is continuous, it is bounded. Since it is bounded it must have a inf and a sup. Since a inf exists in a bounded set I could achrimedian proof it is the minimum. But my professor told me that since the set he wants a minimum in is unbounded my logic wont work. So what am I doing wrong?










share|cite|improve this question











$endgroup$




Let $f:[0,1]to mathbb R$ be continuous and such that $f(0)<0$ and $f(1)=0$. Show that the set $x in (0,1] : f(x)=0 $ has a minimum.



So I though because the function is continuous, it is bounded. Since it is bounded it must have a inf and a sup. Since a inf exists in a bounded set I could achrimedian proof it is the minimum. But my professor told me that since the set he wants a minimum in is unbounded my logic wont work. So what am I doing wrong?







calculus functions






share|cite|improve this question















share|cite|improve this question













share|cite|improve this question




share|cite|improve this question








edited Mar 27 at 19:34









Bernard

123k741117




123k741117










asked Mar 27 at 19:31









JJ BurtJJ Burt

215




215











  • $begingroup$
    The set is closed in $[0,1].$
    $endgroup$
    – Dbchatto67
    Mar 27 at 19:34










  • $begingroup$
    Not in the set im looking for a minimum in?
    $endgroup$
    – JJ Burt
    Mar 27 at 19:38
















  • $begingroup$
    The set is closed in $[0,1].$
    $endgroup$
    – Dbchatto67
    Mar 27 at 19:34










  • $begingroup$
    Not in the set im looking for a minimum in?
    $endgroup$
    – JJ Burt
    Mar 27 at 19:38















$begingroup$
The set is closed in $[0,1].$
$endgroup$
– Dbchatto67
Mar 27 at 19:34




$begingroup$
The set is closed in $[0,1].$
$endgroup$
– Dbchatto67
Mar 27 at 19:34












$begingroup$
Not in the set im looking for a minimum in?
$endgroup$
– JJ Burt
Mar 27 at 19:38




$begingroup$
Not in the set im looking for a minimum in?
$endgroup$
– JJ Burt
Mar 27 at 19:38










3 Answers
3






active

oldest

votes


















1












$begingroup$

The point of the exercise is this:



The set $x in (0,1] : f(x)=0 $ obviously has an infimum, because it is non-empty and bounded below. But is this infimum necessarily a minimum? That is, does the infimum itself belong to the set? This is what you have to prove.



It seems to me that you missed this. It also seems that you must have misunderstood your professor -- what he allegedly said makes no sense to me..



(Also, I presume you mean "archimedean proof". But what do you mean by this?)






share|cite|improve this answer









$endgroup$




















    1












    $begingroup$

    Continuity and $f(0)<0$ implies that there exists a small $epsilon>0$ such that if $0leq x<epsilon$, $f(x) < 0$. Then, the zeros of $f$ are contained in the compact set $[epsilon,1]$. Now let $x_0 = inf_[epsilon,1]left x : f(x) = 0 right$. If this is a finite set, then we are done. If not, consider a sequence from the set converging to $x_0$. Since $f$ is continuous, $f(x_0)=0$ and certainly $x_0in[epsilon,1]$ due to compactness. Hence $x_0$ is the desired minimum.






    share|cite|improve this answer









    $endgroup$




















      0












      $begingroup$

      Hint $:$ $x in (0,1] : f(x) = 0 = f^-1 (0).$ Since $f$ is continuous so $f^-1 (0)$ is a closed subspace of $[0,1]$ (Since $0 $ is a closed subset of $Bbb R$). Since $[0,1]$ is a closed subspace of $Bbb R$ with usual topology so $f^-1 (0)$ is a closed subspace of $Bbb R$ with usual topology.






      share|cite|improve this answer











      $endgroup$












      • $begingroup$
        Inverse image of $0 $ under $f$ is a closed subspace of $[0,1]$ since $0 $ is a closed subspace of $Bbb R$ and $f$ is continuous.
        $endgroup$
        – Dbchatto67
        Mar 27 at 19:41











      • $begingroup$
        Very confused we havnt used much topology in this class so I am not very familiar with it.
        $endgroup$
        – JJ Burt
        Mar 27 at 19:41










      • $begingroup$
        Closed subspace of $[0,1]$ is of the form $A cap [0,1]$ where $A$ is a closed subset of $Bbb R.$ Hence the subsets which are closed in $[0,1]$ must also be closed in $Bbb R$ since intersection of two closed subsets of $Bbb R$ is also a closed subset of $Bbb R.$
        $endgroup$
        – Dbchatto67
        Mar 27 at 19:43











      • $begingroup$
        Do you know @JJ Burt that every closed subset of $Bbb R$ attains it's infimum?
        $endgroup$
        – Dbchatto67
        Mar 27 at 19:55











      • $begingroup$
        that does sound familiar so becuase its a subset?
        $endgroup$
        – JJ Burt
        Mar 27 at 20:00











      Your Answer





      StackExchange.ifUsing("editor", function ()
      return StackExchange.using("mathjaxEditing", function ()
      StackExchange.MarkdownEditor.creationCallbacks.add(function (editor, postfix)
      StackExchange.mathjaxEditing.prepareWmdForMathJax(editor, postfix, [["$", "$"], ["\\(","\\)"]]);
      );
      );
      , "mathjax-editing");

      StackExchange.ready(function()
      var channelOptions =
      tags: "".split(" "),
      id: "69"
      ;
      initTagRenderer("".split(" "), "".split(" "), channelOptions);

      StackExchange.using("externalEditor", function()
      // Have to fire editor after snippets, if snippets enabled
      if (StackExchange.settings.snippets.snippetsEnabled)
      StackExchange.using("snippets", function()
      createEditor();
      );

      else
      createEditor();

      );

      function createEditor()
      StackExchange.prepareEditor(
      heartbeatType: 'answer',
      autoActivateHeartbeat: false,
      convertImagesToLinks: true,
      noModals: true,
      showLowRepImageUploadWarning: true,
      reputationToPostImages: 10,
      bindNavPrevention: true,
      postfix: "",
      imageUploader:
      brandingHtml: "Powered by u003ca class="icon-imgur-white" href="https://imgur.com/"u003eu003c/au003e",
      contentPolicyHtml: "User contributions licensed under u003ca href="https://creativecommons.org/licenses/by-sa/3.0/"u003ecc by-sa 3.0 with attribution requiredu003c/au003e u003ca href="https://stackoverflow.com/legal/content-policy"u003e(content policy)u003c/au003e",
      allowUrls: true
      ,
      noCode: true, onDemand: true,
      discardSelector: ".discard-answer"
      ,immediatelyShowMarkdownHelp:true
      );



      );













      draft saved

      draft discarded


















      StackExchange.ready(
      function ()
      StackExchange.openid.initPostLogin('.new-post-login', 'https%3a%2f%2fmath.stackexchange.com%2fquestions%2f3165005%2fon-continuous-functions%23new-answer', 'question_page');

      );

      Post as a guest















      Required, but never shown

























      3 Answers
      3






      active

      oldest

      votes








      3 Answers
      3






      active

      oldest

      votes









      active

      oldest

      votes






      active

      oldest

      votes









      1












      $begingroup$

      The point of the exercise is this:



      The set $x in (0,1] : f(x)=0 $ obviously has an infimum, because it is non-empty and bounded below. But is this infimum necessarily a minimum? That is, does the infimum itself belong to the set? This is what you have to prove.



      It seems to me that you missed this. It also seems that you must have misunderstood your professor -- what he allegedly said makes no sense to me..



      (Also, I presume you mean "archimedean proof". But what do you mean by this?)






      share|cite|improve this answer









      $endgroup$

















        1












        $begingroup$

        The point of the exercise is this:



        The set $x in (0,1] : f(x)=0 $ obviously has an infimum, because it is non-empty and bounded below. But is this infimum necessarily a minimum? That is, does the infimum itself belong to the set? This is what you have to prove.



        It seems to me that you missed this. It also seems that you must have misunderstood your professor -- what he allegedly said makes no sense to me..



        (Also, I presume you mean "archimedean proof". But what do you mean by this?)






        share|cite|improve this answer









        $endgroup$















          1












          1








          1





          $begingroup$

          The point of the exercise is this:



          The set $x in (0,1] : f(x)=0 $ obviously has an infimum, because it is non-empty and bounded below. But is this infimum necessarily a minimum? That is, does the infimum itself belong to the set? This is what you have to prove.



          It seems to me that you missed this. It also seems that you must have misunderstood your professor -- what he allegedly said makes no sense to me..



          (Also, I presume you mean "archimedean proof". But what do you mean by this?)






          share|cite|improve this answer









          $endgroup$



          The point of the exercise is this:



          The set $x in (0,1] : f(x)=0 $ obviously has an infimum, because it is non-empty and bounded below. But is this infimum necessarily a minimum? That is, does the infimum itself belong to the set? This is what you have to prove.



          It seems to me that you missed this. It also seems that you must have misunderstood your professor -- what he allegedly said makes no sense to me..



          (Also, I presume you mean "archimedean proof". But what do you mean by this?)







          share|cite|improve this answer












          share|cite|improve this answer



          share|cite|improve this answer










          answered Mar 27 at 19:55









          TonyKTonyK

          43.7k358137




          43.7k358137





















              1












              $begingroup$

              Continuity and $f(0)<0$ implies that there exists a small $epsilon>0$ such that if $0leq x<epsilon$, $f(x) < 0$. Then, the zeros of $f$ are contained in the compact set $[epsilon,1]$. Now let $x_0 = inf_[epsilon,1]left x : f(x) = 0 right$. If this is a finite set, then we are done. If not, consider a sequence from the set converging to $x_0$. Since $f$ is continuous, $f(x_0)=0$ and certainly $x_0in[epsilon,1]$ due to compactness. Hence $x_0$ is the desired minimum.






              share|cite|improve this answer









              $endgroup$

















                1












                $begingroup$

                Continuity and $f(0)<0$ implies that there exists a small $epsilon>0$ such that if $0leq x<epsilon$, $f(x) < 0$. Then, the zeros of $f$ are contained in the compact set $[epsilon,1]$. Now let $x_0 = inf_[epsilon,1]left x : f(x) = 0 right$. If this is a finite set, then we are done. If not, consider a sequence from the set converging to $x_0$. Since $f$ is continuous, $f(x_0)=0$ and certainly $x_0in[epsilon,1]$ due to compactness. Hence $x_0$ is the desired minimum.






                share|cite|improve this answer









                $endgroup$















                  1












                  1








                  1





                  $begingroup$

                  Continuity and $f(0)<0$ implies that there exists a small $epsilon>0$ such that if $0leq x<epsilon$, $f(x) < 0$. Then, the zeros of $f$ are contained in the compact set $[epsilon,1]$. Now let $x_0 = inf_[epsilon,1]left x : f(x) = 0 right$. If this is a finite set, then we are done. If not, consider a sequence from the set converging to $x_0$. Since $f$ is continuous, $f(x_0)=0$ and certainly $x_0in[epsilon,1]$ due to compactness. Hence $x_0$ is the desired minimum.






                  share|cite|improve this answer









                  $endgroup$



                  Continuity and $f(0)<0$ implies that there exists a small $epsilon>0$ such that if $0leq x<epsilon$, $f(x) < 0$. Then, the zeros of $f$ are contained in the compact set $[epsilon,1]$. Now let $x_0 = inf_[epsilon,1]left x : f(x) = 0 right$. If this is a finite set, then we are done. If not, consider a sequence from the set converging to $x_0$. Since $f$ is continuous, $f(x_0)=0$ and certainly $x_0in[epsilon,1]$ due to compactness. Hence $x_0$ is the desired minimum.







                  share|cite|improve this answer












                  share|cite|improve this answer



                  share|cite|improve this answer










                  answered Mar 27 at 20:02









                  MatthewPeterMatthewPeter

                  191




                  191





















                      0












                      $begingroup$

                      Hint $:$ $x in (0,1] : f(x) = 0 = f^-1 (0).$ Since $f$ is continuous so $f^-1 (0)$ is a closed subspace of $[0,1]$ (Since $0 $ is a closed subset of $Bbb R$). Since $[0,1]$ is a closed subspace of $Bbb R$ with usual topology so $f^-1 (0)$ is a closed subspace of $Bbb R$ with usual topology.






                      share|cite|improve this answer











                      $endgroup$












                      • $begingroup$
                        Inverse image of $0 $ under $f$ is a closed subspace of $[0,1]$ since $0 $ is a closed subspace of $Bbb R$ and $f$ is continuous.
                        $endgroup$
                        – Dbchatto67
                        Mar 27 at 19:41











                      • $begingroup$
                        Very confused we havnt used much topology in this class so I am not very familiar with it.
                        $endgroup$
                        – JJ Burt
                        Mar 27 at 19:41










                      • $begingroup$
                        Closed subspace of $[0,1]$ is of the form $A cap [0,1]$ where $A$ is a closed subset of $Bbb R.$ Hence the subsets which are closed in $[0,1]$ must also be closed in $Bbb R$ since intersection of two closed subsets of $Bbb R$ is also a closed subset of $Bbb R.$
                        $endgroup$
                        – Dbchatto67
                        Mar 27 at 19:43











                      • $begingroup$
                        Do you know @JJ Burt that every closed subset of $Bbb R$ attains it's infimum?
                        $endgroup$
                        – Dbchatto67
                        Mar 27 at 19:55











                      • $begingroup$
                        that does sound familiar so becuase its a subset?
                        $endgroup$
                        – JJ Burt
                        Mar 27 at 20:00















                      0












                      $begingroup$

                      Hint $:$ $x in (0,1] : f(x) = 0 = f^-1 (0).$ Since $f$ is continuous so $f^-1 (0)$ is a closed subspace of $[0,1]$ (Since $0 $ is a closed subset of $Bbb R$). Since $[0,1]$ is a closed subspace of $Bbb R$ with usual topology so $f^-1 (0)$ is a closed subspace of $Bbb R$ with usual topology.






                      share|cite|improve this answer











                      $endgroup$












                      • $begingroup$
                        Inverse image of $0 $ under $f$ is a closed subspace of $[0,1]$ since $0 $ is a closed subspace of $Bbb R$ and $f$ is continuous.
                        $endgroup$
                        – Dbchatto67
                        Mar 27 at 19:41











                      • $begingroup$
                        Very confused we havnt used much topology in this class so I am not very familiar with it.
                        $endgroup$
                        – JJ Burt
                        Mar 27 at 19:41










                      • $begingroup$
                        Closed subspace of $[0,1]$ is of the form $A cap [0,1]$ where $A$ is a closed subset of $Bbb R.$ Hence the subsets which are closed in $[0,1]$ must also be closed in $Bbb R$ since intersection of two closed subsets of $Bbb R$ is also a closed subset of $Bbb R.$
                        $endgroup$
                        – Dbchatto67
                        Mar 27 at 19:43











                      • $begingroup$
                        Do you know @JJ Burt that every closed subset of $Bbb R$ attains it's infimum?
                        $endgroup$
                        – Dbchatto67
                        Mar 27 at 19:55











                      • $begingroup$
                        that does sound familiar so becuase its a subset?
                        $endgroup$
                        – JJ Burt
                        Mar 27 at 20:00













                      0












                      0








                      0





                      $begingroup$

                      Hint $:$ $x in (0,1] : f(x) = 0 = f^-1 (0).$ Since $f$ is continuous so $f^-1 (0)$ is a closed subspace of $[0,1]$ (Since $0 $ is a closed subset of $Bbb R$). Since $[0,1]$ is a closed subspace of $Bbb R$ with usual topology so $f^-1 (0)$ is a closed subspace of $Bbb R$ with usual topology.






                      share|cite|improve this answer











                      $endgroup$



                      Hint $:$ $x in (0,1] : f(x) = 0 = f^-1 (0).$ Since $f$ is continuous so $f^-1 (0)$ is a closed subspace of $[0,1]$ (Since $0 $ is a closed subset of $Bbb R$). Since $[0,1]$ is a closed subspace of $Bbb R$ with usual topology so $f^-1 (0)$ is a closed subspace of $Bbb R$ with usual topology.







                      share|cite|improve this answer














                      share|cite|improve this answer



                      share|cite|improve this answer








                      edited Mar 27 at 19:52

























                      answered Mar 27 at 19:35









                      Dbchatto67Dbchatto67

                      2,419422




                      2,419422











                      • $begingroup$
                        Inverse image of $0 $ under $f$ is a closed subspace of $[0,1]$ since $0 $ is a closed subspace of $Bbb R$ and $f$ is continuous.
                        $endgroup$
                        – Dbchatto67
                        Mar 27 at 19:41











                      • $begingroup$
                        Very confused we havnt used much topology in this class so I am not very familiar with it.
                        $endgroup$
                        – JJ Burt
                        Mar 27 at 19:41










                      • $begingroup$
                        Closed subspace of $[0,1]$ is of the form $A cap [0,1]$ where $A$ is a closed subset of $Bbb R.$ Hence the subsets which are closed in $[0,1]$ must also be closed in $Bbb R$ since intersection of two closed subsets of $Bbb R$ is also a closed subset of $Bbb R.$
                        $endgroup$
                        – Dbchatto67
                        Mar 27 at 19:43











                      • $begingroup$
                        Do you know @JJ Burt that every closed subset of $Bbb R$ attains it's infimum?
                        $endgroup$
                        – Dbchatto67
                        Mar 27 at 19:55











                      • $begingroup$
                        that does sound familiar so becuase its a subset?
                        $endgroup$
                        – JJ Burt
                        Mar 27 at 20:00
















                      • $begingroup$
                        Inverse image of $0 $ under $f$ is a closed subspace of $[0,1]$ since $0 $ is a closed subspace of $Bbb R$ and $f$ is continuous.
                        $endgroup$
                        – Dbchatto67
                        Mar 27 at 19:41











                      • $begingroup$
                        Very confused we havnt used much topology in this class so I am not very familiar with it.
                        $endgroup$
                        – JJ Burt
                        Mar 27 at 19:41










                      • $begingroup$
                        Closed subspace of $[0,1]$ is of the form $A cap [0,1]$ where $A$ is a closed subset of $Bbb R.$ Hence the subsets which are closed in $[0,1]$ must also be closed in $Bbb R$ since intersection of two closed subsets of $Bbb R$ is also a closed subset of $Bbb R.$
                        $endgroup$
                        – Dbchatto67
                        Mar 27 at 19:43











                      • $begingroup$
                        Do you know @JJ Burt that every closed subset of $Bbb R$ attains it's infimum?
                        $endgroup$
                        – Dbchatto67
                        Mar 27 at 19:55











                      • $begingroup$
                        that does sound familiar so becuase its a subset?
                        $endgroup$
                        – JJ Burt
                        Mar 27 at 20:00















                      $begingroup$
                      Inverse image of $0 $ under $f$ is a closed subspace of $[0,1]$ since $0 $ is a closed subspace of $Bbb R$ and $f$ is continuous.
                      $endgroup$
                      – Dbchatto67
                      Mar 27 at 19:41





                      $begingroup$
                      Inverse image of $0 $ under $f$ is a closed subspace of $[0,1]$ since $0 $ is a closed subspace of $Bbb R$ and $f$ is continuous.
                      $endgroup$
                      – Dbchatto67
                      Mar 27 at 19:41













                      $begingroup$
                      Very confused we havnt used much topology in this class so I am not very familiar with it.
                      $endgroup$
                      – JJ Burt
                      Mar 27 at 19:41




                      $begingroup$
                      Very confused we havnt used much topology in this class so I am not very familiar with it.
                      $endgroup$
                      – JJ Burt
                      Mar 27 at 19:41












                      $begingroup$
                      Closed subspace of $[0,1]$ is of the form $A cap [0,1]$ where $A$ is a closed subset of $Bbb R.$ Hence the subsets which are closed in $[0,1]$ must also be closed in $Bbb R$ since intersection of two closed subsets of $Bbb R$ is also a closed subset of $Bbb R.$
                      $endgroup$
                      – Dbchatto67
                      Mar 27 at 19:43





                      $begingroup$
                      Closed subspace of $[0,1]$ is of the form $A cap [0,1]$ where $A$ is a closed subset of $Bbb R.$ Hence the subsets which are closed in $[0,1]$ must also be closed in $Bbb R$ since intersection of two closed subsets of $Bbb R$ is also a closed subset of $Bbb R.$
                      $endgroup$
                      – Dbchatto67
                      Mar 27 at 19:43













                      $begingroup$
                      Do you know @JJ Burt that every closed subset of $Bbb R$ attains it's infimum?
                      $endgroup$
                      – Dbchatto67
                      Mar 27 at 19:55





                      $begingroup$
                      Do you know @JJ Burt that every closed subset of $Bbb R$ attains it's infimum?
                      $endgroup$
                      – Dbchatto67
                      Mar 27 at 19:55













                      $begingroup$
                      that does sound familiar so becuase its a subset?
                      $endgroup$
                      – JJ Burt
                      Mar 27 at 20:00




                      $begingroup$
                      that does sound familiar so becuase its a subset?
                      $endgroup$
                      – JJ Burt
                      Mar 27 at 20:00

















                      draft saved

                      draft discarded
















































                      Thanks for contributing an answer to Mathematics Stack Exchange!


                      • Please be sure to answer the question. Provide details and share your research!

                      But avoid


                      • Asking for help, clarification, or responding to other answers.

                      • Making statements based on opinion; back them up with references or personal experience.

                      Use MathJax to format equations. MathJax reference.


                      To learn more, see our tips on writing great answers.




                      draft saved


                      draft discarded














                      StackExchange.ready(
                      function ()
                      StackExchange.openid.initPostLogin('.new-post-login', 'https%3a%2f%2fmath.stackexchange.com%2fquestions%2f3165005%2fon-continuous-functions%23new-answer', 'question_page');

                      );

                      Post as a guest















                      Required, but never shown





















































                      Required, but never shown














                      Required, but never shown












                      Required, but never shown







                      Required, but never shown

































                      Required, but never shown














                      Required, but never shown












                      Required, but never shown







                      Required, but never shown







                      Popular posts from this blog

                      Triangular numbers and gcdProving sum of a set is $0 pmod n$ if $n$ is odd, or $fracn2 pmod n$ if $n$ is even?Is greatest common divisor of two numbers really their smallest linear combination?GCD, LCM RelationshipProve a set of nonnegative integers with greatest common divisor 1 and closed under addition has all but finite many nonnegative integers.all pairs of a and b in an equation containing gcdTriangular Numbers Modulo $k$ - Hit All Values?Understanding the Existence and Uniqueness of the GCDGCD and LCM with logical symbolsThe greatest common divisor of two positive integers less than 100 is equal to 3. Their least common multiple is twelve times one of the integers.Suppose that for all integers $x$, $x|a$ and $x|b$ if and only if $x|c$. Then $c = gcd(a,b)$Which is the gcd of 2 numbers which are multiplied and the result is 600000?

                      Ingelân Ynhâld Etymology | Geografy | Skiednis | Polityk en bestjoer | Ekonomy | Demografy | Kultuer | Klimaat | Sjoch ek | Keppelings om utens | Boarnen, noaten en referinsjes Navigaasjemenuwww.gov.ukOffisjele webside fan it regear fan it Feriene KeninkrykOffisjele webside fan it Britske FerkearsburoNederlânsktalige ynformaasje fan it Britske FerkearsburoOffisjele webside fan English Heritage, de organisaasje dy't him ynset foar it behâld fan it Ingelske kultuergoedYnwennertallen fan alle Britske stêden út 'e folkstelling fan 2011Notes en References, op dizze sideEngland

                      Հադիս Բովանդակություն Անվանում և նշանակություն | Դասակարգում | Աղբյուրներ | Նավարկման ցանկ